1
$\begingroup$

I've encountered some problem which seems general enough to have already been solved.

There is a set of objects $O=\{o_1, o_2,\dots,o_k\}$ and a family of sets $A_1,A_2,\dots,A_t \subseteq O$.

For every 1ドル \leq j < k$ we need to find a subset $O' \subset O$ of size $j$ maximizing the number of sets $A_i$ contained in it.

What is the best algorithm to solve this? I've been thinking about reducing it to a problem in graphs or flow networks but still haven't arrived at a solution.

Yuval Filmus
281k27 gold badges317 silver badges514 bronze badges
asked Mar 30, 2020 at 17:48
$\endgroup$
1
  • $\begingroup$ Possibly Related: From "covering designs", and assuming we can pick the sets, we arrive at Steiner systems in the best case. You can see Dan Gordon's page if this is what you're after here: dmgordon.org/cover $\endgroup$ Commented Aug 23, 2021 at 3:08

1 Answer 1

1
$\begingroup$

Your problem is essentially Minimum $k$-Union. In this problem (switching from $k$ to $\ell$), you want to find $\ell$ sets out of $A_1,\ldots,A_t$ which together cover the least number of elements. Denoting by $f(\ell)$ the solution of this problem and by $g(j)$ the solution of your problem, we have $$ f(\ell) \leq j \Longleftrightarrow \ell \leq g(j). $$ In other words, $g(j)$ is the maximal $\ell$ such that $f(\ell) \leq j$.

Unfortunately, Minimum $k$-Union is NP-hard. You can find some links in this question on StackOverflow.

answered Mar 30, 2020 at 18:07
$\endgroup$
1
  • $\begingroup$ Thanks for the reply. I see the similarity, the problem is I want to find the solution for every 1ドル \leq j < k$. I'd like to use an approximated solution to the minimum k-union, but to solve for every j I'd need to apply the solution for $ 1,2,...,numof matches=M $. since $M$ can be as large as 2ドル^k$ that's not very good. If the processing time of the solution is $T$ then a total time of 2ドル^k * T$ will be needed, whereas if there is a solution which its input is the number of objects from $O$ and which takes time $R,ドル a total time of $k*R$ is needed, assuming $T \approx R$ this is better. $\endgroup$ Commented Apr 1, 2020 at 18:36

Your Answer

Draft saved
Draft discarded

Sign up or log in

Sign up using Google
Sign up using Email and Password

Post as a guest

Required, but never shown

Post as a guest

Required, but never shown

By clicking "Post Your Answer", you agree to our terms of service and acknowledge you have read our privacy policy.

Start asking to get answers

Find the answer to your question by asking.

Ask question

Explore related questions

See similar questions with these tags.